Foro de preguntas y respuestas de matemáticas, de cualquier nivel. Cuánto más interesantes, divertidas o intrépidas, mejor.
Aviso: Te invitamos a conocer la página de Facebook de la UCIM

Ganas puntos al hacer preguntas, contestarlas y, sobre todo, si tu respuesta es seleccionada como la mejor.
Registrate como usuario para participar en el foro. También puedes utilizar tu identidad de FB Utiliza el botón azul para ingresar (si usas tu identidad de FB y estás logeado en FB, automáticamente te reconoce).

El irracional tiene una página en FB. El Irracional






+2 votos
Hola, les agradecería si alguien me puede orientar para construir un conjunto medible que no sea de Borel y la justificación de por que no es de Borel, gracias de antemano
por (260 puntos) en Preguntas

2 Respuestas

+2 votos
 
Mejor respuesta

Cualquier conjunto Lebesgue medible se puede escribir como diferencia simétrica de un conjunto de Borel y un conjunto de medida cero, así que esencialmente puedes restringir tu búsqueda a conjuntos de medida 0. Así no es díficil probar que hay conjuntos medibles que no son de Borel: toma un conjunto $X$ de medida cero y cardinalidad del continuo (un conjunto de Cantor estándar, por ejemplo); el número de conjuntos de Borel es la cardinalidad del conjunto, pero el número de subconjuntos de $X$ es mayor que eso, así que $X$ debe tener subconjuntos que no son de Borel (y claro, cualquier subconjunto de $X$ también tiene medida $0$).

Pero tu dijiste "construir" y nunca sé que tan en serio tomarme esa palabra, no todo mundo la usa igual. Si de veras quieres un ejemplo explícito de un conjunto así, el que recuerdo haber oído lo construyó Lusin (una búsqueda rápida en Internet me hace pensar que fue en este artículo de 1927 que se llama "Sur les ensembles analytiques", publicado en Fundamenta Mathematicae). El ejemplo era el conjunto de reales que tienen una expansión como fracción continua $$a_0 + \cfrac{1}{a_1 + \cfrac{1}{a_2 + \cfrac{1}{\cdots}}}$$ tal que, para alguna sucesión de enteros postivos $r_1 < r_2 < \cdots$, sucede que $a_{r_i}$ es divisor de $a_{r_{i+1}}$. Lusin demostró que este conjunto no es de Borel pero que sí es analítico y por lo tanto Lebesgue medible. Creo que la intuición de porqué no es de Borel es que dada una sucesión fija $r_1 < r_2 < \cdots$, los números con una expansión de la forma pedida sí son un conjunto de Borel, pero la cantidad de sucesiones $r_1 < r_2 < \cdots$ es la cardinalidad del continuo y una unión de Borelianos con tantos uniendos usualmente no es de Borel.

por (33,2m puntos)
seleccionada por
+3 votos

"Prefiero una prueba clara de existencia a

una construcción con $2^{2^{100}}$ pasos"

- Emil Artin

Otra manera de contestar tu pregunta Memo es demostrar que hay MUCHOS más conjuntos Lebesgue medibles que conjuntos Borel medibles (al menos en los reales).

La familia de borelianos $\mathcal B$ se describe como la $\sigma$-álgebra generada por la topología del espacio $X$. Si $\omega_1$ es el primer ordinal no numerable, entonces $\mathcal B$ se expresa como sigue:

$$\mathcal B=\bigcup_{\alpha\in \omega_1}\Sigma^{0}_{\alpha}=\bigcup_{\alpha\in\omega_1}\Pi^{0}_{\alpha},$$

donde las familias $\Sigma^{0}_{\alpha}$ y $Pi^{0}_{\alpha}$ se definen de manera recursiva: $\Sigma^{0}_{1}=\tau$, donde $\tau$ es la topología del espacio, $\Pi^{0}_{1}=\{X\backslash U:U\in\Sigma^{0}_1\}$, para $\alpha\in\omega_1$ ordinal no límite,

$$\Sigma^{0}_{\alpha}=\left\{\bigcup_{i\in\omega}F^{i}_{\alpha}:(\forall i\in\omega)(F^{i}_{\alpha}\in\Pi^{0}_{\alpha-1})\right\}$$

y si $\alpha$ es límite, definimos

$$\Sigma^{0}_{\alpha}=\left\{\bigcup_{i\in\omega}F^{i}_{\alpha}:(\forall i\in\omega) \left(F^{i}_{\alpha}\in\bigcup_{\beta<\alpha}\Pi^{0}_{\beta}\right)\right\};$$

Las familias $\Pi^{0}_{\alpha}$ siempre se definen como los complementos de la correspondiente familia $\Sigma^{0}_{\alpha}$. Ahora, si $X=\mathbb R$, podemos probar, sin mucha dificultad, que la cardinalidad de cada familia es $\mathfrak c$, el cardinal del continuo. Como $\aleph_1\leq\mathfrak c$, entonces la cardinalidad de los borelinaos es $\mathfrak c$. Sin embargo, si $C$ es el conjunto de Cantor, entonces $\mathcal P(C)$ es una familia de conjuntos medibles y se tiene así que la cardinalidad de la familia de conjuntos Lebesgue medibles de $\mathbb R$ es $2^{\mathfrak c}$. Por el Teorema de Cantor, existe un subconjunto medible, inclusive dentro del Cantor, que no es Borel medible.

por (9,2m puntos)
Mencioné este argumento en el primer párrafo de mi respuesta, pero no expliqué cómo probar que la cantidad de Borelianos es la cardinalidad del continuo; gracias por escribir sobre cómo se prueba eso.
Simón, hacía rato que no aportaba. Ya era hora, ¿no? jajajaja
Licencia Creative Commons
Este obra está bajo una Licencia Creative Commons Atribución-NoComercial-CompartirIgual 2.5 México.

powered by UCIM  -  Aviso de privacidad

...